Finding Max/Min Values on Functions of 3 Variables, Bounded by Ellipsoids

Click For Summary
The discussion focuses on finding the absolute minimum and maximum values of the function F(x,y,z) = x² - 2x - y² + z², constrained by the ellipsoid G(x,y,z) = x² + 4y² + z² = 4. The approach involves using Lagrange multipliers to determine critical points, leading to equations that suggest potential values for x, y, and z. Evaluating these points reveals that the maximum occurs at (-2,0,0) with a value of 8, while the minimum occurs at (4/5, ±2√21/5, 0) with a value of -9/5. The calculations confirm the identified extrema, validating the solution process.
TranscendArcu
Messages
277
Reaction score
0

Homework Statement


Find the absolute minimum and maximum of F(x,y,z) = x2 - 2x - y2 + z2 on the ellipsoid G(x,y,z) = x2 + 4y2 + z2 = 4

Homework Equations


The Attempt at a Solution


I was thinking of trying to solve this by using Lagrange multipliers. So, finding the gradients:

Fx = 2x - 2 = Gx = λ 2x
Fy = - 2y = Gy = λ 8y
Fz = 2z = Gz = λ 2z

From the first partial derivative I have 2x - 2 - λ2x = 0, which suggests x = 1/(1-λ). From the second partial derivative I have y(-2 - λ * 8) = 0, which suggests y = 0. Similarly, from the third partial derivative I have z(2 - λ * 2) = 0, which suggests z = 0. From G(1/(1-λ),0,0) I get λ = ((-/+) 1/2) + 1, or λ = 1/2 or 3/2.

Therefore, x = -2 or 2. Evaluating F(-2,0,0) = 8 and F(2,0,0) = 0. So, (-2,0,0) is our max and (2,0,0) is our min.

Does that sound about right?
 
Physics news on Phys.org
The two equations may suggest a variable is 0 but it doesn't follow that it must be! However, you can argue that if y is not 0, then, dividing both sides of the second equation by y, \lambda= -1/4. Similarly, if z is not 0, \lambda= 1. If \lambda= 1, the first equation cannot be satisfied but if \lambda= -1/4 the first equation gives x= 4/5. Putting x= 4/5, z= 0 into x^2+ 4y^2+ z^2= 4 gives 16/25+ 4y^2= 4 so y^2= 84/25 and then y= 2\sqrt{21}/5. (4/5, 2\sqrt{21}/5, 0) also satisfies those equations. What is F(4/5, 2\sqrt{21}/5, 0)?
 
Last edited by a moderator:
HallsofIvy said:
The two equations may suggest a variable is 0 but it doesn't follow that it must be! However, you can argue that if y is not 0, then, dividing both sides of the second equation by y, \lambda= -1/4. Similarly, if z is not 0, \lambda= 1. If \lambda= 1, the first equation cannot be satisfied but if \lambda= -1/4 the first equation gives x= 4/5. Putting x= 4/5, z= 0 into x^2+ 4y^2+ z^2= 4 gives 16/25+ 4y^2= 4 so y^2= 84/25 and then y= 2\sqrt{21}/5. (4/5, 2\sqrt{21}/5, 0) also satisfies those equations. What is F(4/5, 2\sqrt{21}/5, 0)?

Do you mean y = ± √(21)/5, because 16/25 + 4(2sqrt(21)/5)^2 ≠ 4. Anyway, presuming this is what you meant, I have to test the points (2,0,0), (-2,0,0), (4/5,sqrt(21)/5,0), (4/5,-sqrt(21)/5,0).

F(2,0,0) = 0
F(-2,0,0) = 8
F(4/5,sqrt(21)/5,0) = -9/5
F(4/5,-sqrt(21)/5,0) = -9/5

Therefore, my max is at (-2,0,0) and is 8. My minimums are at (4/5,sqrt(21)/5,0) and (4/5,-sqrt(21)/5,0) and are -9/5.

Sound about right?
 
Question: A clock's minute hand has length 4 and its hour hand has length 3. What is the distance between the tips at the moment when it is increasing most rapidly?(Putnam Exam Question) Answer: Making assumption that both the hands moves at constant angular velocities, the answer is ## \sqrt{7} .## But don't you think this assumption is somewhat doubtful and wrong?

Similar threads

  • · Replies 7 ·
Replies
7
Views
1K
Replies
2
Views
2K
  • · Replies 11 ·
Replies
11
Views
2K
Replies
6
Views
2K
Replies
4
Views
2K
Replies
5
Views
2K
  • · Replies 11 ·
Replies
11
Views
2K
  • · Replies 3 ·
Replies
3
Views
1K
  • · Replies 2 ·
Replies
2
Views
2K
Replies
10
Views
1K